0

Possible Duplicate:
Basic buoyancy question: Man in a boat with a stone

If you have a large boulder on a boat, in a pond, and you throw the boulder overboard and into the pond, would the water level decrease, increase, or remain the same?

I believe that the level of water would remain constant because the same amount of force is acting on the water regardless of the rocks position, thus the system is displacing the same amount of water.

  • I submit that this question is better in quality and should not be closed, because it poses a hypothesis, as opposed to just wanting an answer. – OneChillDude Dec 18 '12 at 16:43
  • You wrote "because the same amount of force is acting on the water regardless of the rocks position". That's wrong. This is included in my answer to a previous question. If the rock is thrown over, the ground on the bottom of the lake exerts a force on the rock. So no, the total force acting on the water is not the same. – Alan Rominger Dec 18 '12 at 16:46
  • @AlanSE - oops yes, I remember your answer to this but couldn't find the question. That question would be a better candidate as a duplicate. – John Rennie Dec 18 '12 at 16:49
  • Hmm, I see your point. Should I delete this question? – OneChillDude Dec 18 '12 at 16:49
  • You can delete it if you want, and if you don't it will probably be closed by us, it's nothing personal. @JohnRennie out of the two candidates either should suffice, since this doesn't require generalization to buoyant and sinking objects. I wonder if some day someone will ask about the water level as the rock falls through the lake at terminal velocity. – Alan Rominger Dec 18 '12 at 16:52
  • 1
    @AlanSE Why was this closed as an exact duplicate without showing the link to the duplicate question? Isn't that the usual procedure? – FrankH Dec 18 '12 at 18:07
  • @FrankH I identified a link when I cast my vote, so I actually don't know. May be a good thing to ask on meta. – Alan Rominger Dec 18 '12 at 18:16
  • @alanse please just post the duplicate link here- I assume the last vote to close would show the cause since each of the three votes could have different reasons. – FrankH Dec 19 '12 at 16:25
  • original: http://physics.stackexchange.com/questions/43051/dropping-an-anchor-from-a-boat also closed as duplicate to the above one, but more general: http://physics.stackexchange.com/questions/41672/a-man-on-a-boat-with-something-in-his-hand – Alan Rominger Dec 19 '12 at 16:42

0 Answers0